Discussion

If T's appointment is immediately after P's appointment and immediately before W's appointment, then which one of the following must be true?
(A)W's appointment is at some time before R's appointment.
(B)...
(C)...
(D)...
(E)...
(F)...
*This question is included in Sequencing: Lesson Set 1 (of 5) - Intro, question #5

The solution is

Posted: 10/04/2012 23:50
It seems like it is possible to have a sequence that progresses as follows: U P T Q S R W
in this case, P is before R and answer D would not be the case. (?)
Image Not Available
Contributor
Posted: 10/05/2012 15:02
P Q R S T U V W

from the statements, we can arrange one scenario when T has slot 3:
__ _P_ _T_ _W_ ___ ___ ___
1 2 3 4 5 6 7

Since Q < W and U < P, this scenario is not feasible, because we can't fit Q and U both in slot 1.

The other scenarios are if R has appointment 3:
__ _S_ _R_ ___ ___ ___ ___
1 2 3 4 5 6 7

__ ___ _R_ _S_ ___ ___ ___
1 2 3 4 5 6 7

In either sub-case, we have to be able to fit the sequence P-T-W, taking up 3 slots. That is, PTW must be after R or after RS, occupying any 3 slots in 4-5-6-7 in the first sub case, or 5-6-7 in the 2nd sub-case.

So, D.

Posted: 10/05/2012 20:32
Thanks for your response. However, I am not clear as to why W must appear immediately after P and T. Q and U don't need to come before both W and P.
Thus, if the order in this case is:
U P T R S Q W
it seems all the rules are being followed.
PT form a block as is stipulated by the question.
U comes before P
Q comes before W
T takes up appointment 3
And SR form a block.
This allows P to come before R.

I'm most likely missing something. Can you help point it out? Thanks.
Image Not Available
Contributor
Posted: 10/05/2012 20:40
Jina, you are missing the statement right before the choices, which explicitly says: "If T's appointment is immediately after P's appointment and immediately before W's appointment, ...", hence we are constrained by an additional condition that P-T-W needs to appear consecutively.

In problems like this (sequencing), it is best to diagram the scenarios like I did, including the sub-scenarios, so you can rule out the invalid ones, and zoom in on the feasible ones.
Posted: 10/05/2012 20:42
I completely missed that. Thanks Joel.
Posted: 11/17/2012 15:56
Can you help me understand why U doesn't have to be prior to R? If the order P,T,W must hold then both Q and U must be ahead of that group. (Q before W and U before P) If that's true, S can't occur ahead of R because you'd need 3 letters, S,Q, and U before R and only 2 spots.

U or Q, R, S, P, T, W

U before R & R before P?
Image Not Available
Contributor
Posted: 11/18/2012 04:07
Carl,
First we'll set the conditions into definitions:
Def. 1 Q < W
Def. 2 U < P
Def. 3 ( R || T ) == 3rd
Def. 4 ( SR ) || ( RS )
Def. 5 ( PTW )

We have 7 slots AND 7 different symbols to set, first we look at our definitions for constants with respect to position, definition 3 targets constant location, thus:
1 2 3 4 5 6 7
- - R
- - T
Def. 5 takes three consecutive slots this limits the placement to:
- P T W
- - R P T W
- - R - P T W
Def. 1 and 2 force the 5th definition PTW to have Q and U before it thus T as 3rd is not an option since there's only one slot in front, so now we have:
1 2 3 4 5 6 7
- - R P T W
- - R - P T W
Now we implement def. 4 ( SR || RS )
- S R P T W - >>> only one spot before PTW thus no option
- S R - P T W
- - R S P T W
Thus the valid sequences are:
seq. 1 U S R Q P T W
seq. 2 Q S R U P T W
seq. 3 U Q R S P T W
seq. 4 Q U R S P T W

Answer A. W < R always false
Answer B. U < R see seq. 2 thus false
Answer C. S < U see seq. 1, 3 and 4 thus false
Answer D. R < P always true
Answer E. Q < S see seq. 1 thus false

Niels

You need to be signed in to perform that action.

Sign In